Вы находитесь на странице: 1из 15

Revision questions of previous modules:

1. Mr. A had taken a loan of Rs. 40 lakh in July 2010 at a floating rate of interest
of 10% p.a for tenure of 20 years from a housing finance company. The
company sent a notice raising the interest rate to 10.75% p.a. effective
January 2012 thereby increasing EMI. He decides to refinance the loan at
10.25% from a bank which charges a processing fee of 1% of loan sanctioned.
What absolute amount he stands to save in the remaining tenure if the
outstanding loan amount as at end of March 2012 is refinanced so that the
new loan terminates as per original tenure?
Solution:

Tenure of loan 240 Months


Loan amount 4,000,0 Rs.
00
Initial Rate of 10.00% p.a.
Interest
EMI began in July 38,601 Rs. S=en
2010 d,n=20*12,i=10,pv=-
40lac,pmt=?,fv=0,p/y=1
2,c/y=12
EMI installments repaid till 18
December 2011
Loan outstanding 3,898,1 Rs. Pm1=1,pm2=18,bal=sol
as at December 60 ve
2011
Revised Rate of 10.75% p.a.
Interest beginning
January 2012
New EMI effective 40,516 Rs. Set=end,n=222-
January 2012 18,i=10.75,pv=-
3898160,pmt=?,fv=0,p/
y=12,c/y=12

Loan outstanding to 3,881,2 Rs. Pm1=1,pm3=3,bal=solv


be repaid to 26 e
Finance Co. in Mar
12
Processing fee @ 38,812 Rs. 3881226*1%
1% of outstanding
loan taken fr. Bank
Rate of Interest 10.25% p.a.
charged by bank
beginning April
2012
Outstanding tenure 219 mont 240-18-3
being new tenure hs
for bank loan
EMI to be charged 39,245 Rs. S=end,n=240-18-
by Bank 3,i=10.25,pv=-
3881225,pmt=?,fv=0,p/
y=12,c/y=12
Amount to be 8,872,9 Rs. 40516*219
incurred from Apr 08
2012 in earlier loan
Revised amount 8,594,6 Rs. 39245*219
incurred towards 94
EMIs
Processing fees 8,633,5 Rs. 8594655+38812
included in revised 07
amount incurred
Savings (absolute) 239,40 Rs. 8872908-8633507
in refinancing the 1
loan

2. A familys monthly expenditure is Rs. 40,000. The earner accounts for 15% of
the expense. He wants to cover his familys inflation adjusted expenses for
the next 40 years considering average inflation at 5.5% p.a. and the
investment return at 7.5% p.a. The approximate life insurance needed
is____________

Solution:
Calculation of insurance required
Particulars Amount
(INR)
PV (Net monthly family expenses) 1,14,84,27
(WNo. 1 & 2) 3.3

WNo. 1:- Calculation of Net Family Expenses


Particulars Amount
(INR)
Gross monthly family expenditure 40,000
(-) Expenses of the earner (15% of Rs. (6,000)
40,000)
Net monthly family expenditure 34,000

WNo. 2:- Calculation of the Amount of Insurance required


Particulars Amount (INR)
Set Begin
N = No. of payment periods 40 x 12 = 480
I% = real rate of return = [(1.075/1.055)- 1.895734597
1] x 100
PV = Amount of insurance required ? (Solve =
1,14,84,273.3)
PMT = Net monthly family expenditure - 34,000
(WNo. 1)
FV, P/Y & C/Y 0,12,1 respectively

3. Your client is 32 years old & has Rs. 25,00,000 in loan liabilities. He has a non
working spouse of age 30 & children of age 7 years & 5 years. Additionally,
the client wants higher education for each of his children to the tune of Rs.
30,00,000 after 15 years & marriage expenses of Rs. 15,00,000 after 20 years
(both are considered at current costs). The present household expenses are
Rs. 50,000 per month, which includes housing loan EMI of Rs. 15,000. The
client consumes, Rs. 8,000 per month as personal expenses. He also wants to
provide for 50 years living expenses for his spouse. He has an insurance cover
of Rs. 40,00,000 presently & his financial investments are Rs. 15,00,000. The
additional quantum of life insurance cover is ____________ (Assume:- All
expenses required are inflation adjusted at an average inflation of 5% p.a. &
the claim amount invested to yield is 8% p.a.)
Solution:

Amount of additional life insurance required


Particulars Amount (INR)
PV (Loan) (given) 25,00,000
(+) PV (higher education for both children) (WNo. 1 39,32,189.184
& 2)
(+) PV (marriage expenses) (WNo. 3 & 4) 17,07,780.799
(+) PV (living exp for spouse) (WNo. 5 & 6) 86,99,400.523
(-) Insurance cover already taken (given) (40,00,000)
(-) Available investments (given) (15,00,000)
Additional life insurance cover required 1,13,39,370.51

WNo. 1:- Cost of higher education


after 15 years
Particulars Amount (INR)
Set Begin
N 15
I% = inflation 5
rate
PV - 60,00,000
PMT 0
FV ? (Solve =
1,24,73,569.08)
P/Y & C/Y 1
WNo. 2:- Amount to be invested
today to achieve the education cost
after 15 years
Particulars Amount (INR)
Set Begin
N 15
I% = investment 8
rate
PV ? (Solve = -
39,32,189.184
PMT 0
FV 1,24,73,569.08
P/Y & C/Y 1

WNo. 3:- Cost of marriage


expenses after 20 years
Particulars Amount (INR)
Set Begin
N 20
I% = inflation 5
rate
PV - 30,00,000
PMT 0
FV ? (Solve =
79,59,893.115)
P/Y & C/Y 1
WNo. 4:- Amount to be invested
today to achieve the cost of
marriage after 20 years
Particulars Amount (INR)
Set Begin
N 20
I% = investment 8
rate
PV ? (Solve = -
17,07,780.799)
PMT 0
FV 79,59,893.115
P/Y & C/Y 1

WNo. 5:- Net household expenses


(per month)
Particulars Amount (INR)
Present household 50,000
exp
(-) EMI (loan (15,000)
amount shown
separately)
(-) Personal exp (8,000)
Net household 27,000
exp
WNo. 6:- PV of living expenses
Particulars Amount (INR)
Set Begin
N 50 x 12 = 600
I% = real rate of 2.857142857
return =
[(1.08/1.05)-1] x
100
PV ? (Solve =
86,99,400.523)
PMT - 27,000
FV, P/Y , C/Y 0, 12 , 1 (resp)

4. The factory was made 6 years ago for 80 lakhs. The construction cost is
increasing at 8% per annum. Depreciation is at 6%/annum using Straight Line
Method. What value is likely to be insurable under fire on Market Value Basis
Solution:
S B
N 6
I 8
Pv -8olacs
Pmt O
Fv ?12694994
p/y 1
c/y 1
12694994-4590197(1264994*6%*6)=3305204
5. A company XYZ paid dividends of Rs. 2 recently on its common stock.
Dividend is expected to grow by 20% year on year for the next 3 years after
which it is expected to stabilize at 10% growth. If your requirements of the
return from company stock is 15%. What is the maximum price that you
would pay today to buy the stock?
Solution:
Do 2
D1 2.4 (2*1.2)
D2 2.88 (2.4*1.2)
(2.88*1.2
D3 3.456 )

STEP 1
Find NPV in calculator for cash flow below
1 0
2 2.4
3 2.88
4 3.456
NPV Rs. 6.54

STEP 2
Use Formula:
(D*(1+G))/(R-G)

therefore, NPV of all future cash 76.032


flow of dividends

STEP 3
Find PV of above figure
Rs. 76.032 is NPV as on 4th year so find its
PV as on date
n 4
i 15%
Rs.
PV -43.47
PMT 0
FV 76.032

Hence, Intrinic value of Rs.


share as on date is 50.01
6.54+43.47

6. A 10 year 8% corporate Bond with FV of 1000 and interest payable


semi-annually, matures after 5 years. It Yield to Maturity is 6% p.a.
Record date for coupon just passes away. At what vale 100 bonds of
the corporate are likely to be quoted in market.
Solution:
Set End
N 10
I 0.03(6%/2)
Rs.
=pv(PV Solve 108,530.20
((100*1000)*
PMT 4000 8%)/2
FV 100000 100*1000
P/Y 1
C/Y 1

7. An individual start investing immediately for 10 year annually Rs 80000 in the


ratio 70:30 in equity and debt products. He expects the return from equity and
debt to be 12.5% pa & 9.5% pa. during this period. To protect the wealth he
rebalance the portfolio in 40:60 of equity and debt after 10 yrs and invest in
same ratio annually rs 1.5 lakh for next 10 years. The return expected from
equity and debt in this period subsides to 10.5% pa and 7.5% pa respectively.
What could be his total investment at the end of the entire tenure of his
investment?
Solution: first 10 years, investment amount = 80000(p.a.)
Particulars Equity Debt
Allocation 70 30
N 10 10
I 12.5 9.5
Pmt -56000(80000*70%) -24000(80000*30%)
Fv (solve) 1132649.7 408924

Next 10 years, investment amount = 150,000(p.a.) + FV total of 1 st 10 years =


1541573
Particulars Equity Debt
Allocation 40 60
N 10 10
I 10.5 7.5
Pv - -
616629.37(1541573*40% 924944.06(1541573*60%
) )
Pmt -60000(150000*40%) -90000(150000*60%)
Fv (solve) 275501.58 3275069.623
FV total of next 10 years = 6030971.206

8. A businessman sold Rs. 85 lakh value of unlisted securities on 20th December


2015. These shares were acquired in April 2011 for Rs. 20 lakh. He invested
Rs. 40 lakh from these proceeds in February 2016 in his first residential house
to avail benefit under Section 54F of the Income-tax Act, 1961. What
approximate amount of bonds specified under Section 54EC should he
purchase and by what date so as to make his capital gains liability almost Nil
towards these transactions? Cost inflation index for FY 2011-12 : 785 , 2015-
16 : 1081.
Solution: FVC=8500000
Less: Expense=0
NVC= 8500000
8500000-1452358 (ICOA=20l*785/1081)=7047641-
3316537(exemption=40L/85L*7047641)=3731104LTCG-3731104=0, 20/12/15
invst date(after 6mnths)
9. An investor purchased 2,000 shares of a listed company at Rs. 125 per share
on 28th August 2014. The Company declared a dividend of Rs. 8 per share,
the record date was 26th November 2014. He sold 900 shares on 12th
February, 2015 at a price of Rs. 113 per share and the balance on 27th June
2015 at a price of Rs. 135 per share. He had no other transactions during FY
2014-15. What is the disposition of his sell transactions for AY 2016-17?
Solution: 14-15= (113-125)*900=(10800)STCl
Dividend=8*900=7200
(10800)-7200=(3600)loss set of nxt yr
14-15=(135-125)*100=11000
11000-(3600)=7400

10.Municipal value of a house is Rs 1lakh, Rent 1, 50,000, standard Rent Rs 1,


20,000. The house property has let out for Rs 12,000 per month and vacant
for 1 month during previous year 2015-16. Municipal tax paid by owner Rs
4,500 in previous year. What will be the income from house property?
Solution:
Particulars Amount
Municipal Value 100000
Fair Rent 150000
Expected Rent 150000
Standard Rent 120000
Reasonable let out value 120000
Actual rent (pa) Unrealised rent (cy) 144000 (12000*12)-0
Less: Vacancy Loss 12000 (12000*1)
GAV 132000 (144000-12000)
Less: Municipal taxes 4500
NAV 127500
Less: Deductions
30% of NAV 38250
Income from house property 89250

11.You advise your client aged 31 years to accumulate corpus of for retirement.
The client already has in Balanced MF scheme Rs. 1.60 lakh which you advise
to extend to achieve this goal. You advise him to start SIP of Rs. 5,000 every
month till his age of 35 years, thereafter increase the same to Rs. 7,500 p.m.
till his age 40 years, Rs. 10,000 p.m. between 40 50 years, and Rs. 15,000
p.m. between 50 56 years. You advise him to switch 25% of outstanding
Balanced MF portfolio every year to Liquid schemes from age 57 until full
redemption on retirement at age 60. How much of the retirement corpus
would he be able to accumulate? (Rate of return Balanced MF 9% p.a., Liquid
MF 5.5% p.a.)
12.Your client aged 34 now requires at his retirement age of 60 years a corpus to
sustain an annuity of Rs. 55,000 p.m. (current cost) inflation linked for a post-
retirement life of 25 years up to which he expects to live. You estimate that his
goal would be achieved by investing corpus at a return of 8%. Your client
apprises you that he would additionally like to start a Trust with a donation of
Rs. 1 crore (value then) on his reaching age 70 years and would bequeath
posthumously a further amount of Rs. 1 crore (value then) for his son. He asks
you whether this arrangement would be feasible by taking a little more risk
while investing the retirement corpus. You estimate by taking 1% additional
return than envisaged and opine that ______. (Consider inflation at 5.5%)
13.The higher education costs per annum are Rs. 3 lakh at present costs. The
costs are escalating @9% per annum. Mr. A estimates for his son that such
funds would be required for 5 years after 6 years from now. He starts
accumulating funds immediately in a systematic manner every month in an
equity mutual fund scheme. He would switch equivalent funds required for a
particular year to liquid mutual fund scheme one year in advance. The funds
would continue to be accumulated for a period up to the last switch to liquid
fund. What should be the SIP amount in the equity growth fund? (Take
expected return from equity growth funds @12% p.a. and from liquid fund
@6% p.a.)

Questions (Guidance notes by FPSBI)

Problem Type 1:
Situation:
Current household expenses of a couple 50,000 Rs. p.m.
Current age of Earning member 35 years
Current age of dependent spouse 32 years
Retirement age of Earning member 58 years
Life expectancy of Earning member 75 years
Life Expectany of dependent spouse 80 years
12,000,0
Accumulation from existing investments 00 Rs.
Goal:
They would require regular monthly inflation-linked stream of income equivalent to their
current household expenses till the life of Earning member and thereafter 60% of that
income till the spouse survives. The corpus is supposed to be invested at 7% p.a. What
additional monthly amount to be invested with immediate effect in an
investment yielding 11% p.a. to attain the desired corpus? (given inflation
throughout is 5.5% p.a.)

Problem Type 2:
Situation:
Today's date is 1st April, 2013:
Current age of First Child 4 years
Current age of Second Child 1 year
Goal:
Education expenses are required for each child at their respective age of 18 (Rs. 4 lakh at
current cost) and for four subsequent years (Rs. 3 lakh p.a. at current cost). Expenses
escalate at 5.5% p.a. All withdrawals are made in the beginning of the financial year.
What monthly amount is to be invested at 11% p.a. with immediate effect up to
one year prior to the required expenses for the First Child to achive this goal?

Problem Type 3:
A personal loan of Rs. 3 lakh is availed on credit card at 14% p.a. interest for tenure of 2
years. The credit card company charged processing fees of 1% of the loan amount. The
interest on monthly reducing balance basis was charged in the credit card. What is the
annual effective rate of interest paid in this transaction?

Problem Type 4:
Situation:
Employee's Gross salary per 1,000,0 Rs.
annum 00
Estimated tax during the year 210,00 Rs.
0
Family's monthly expenses 25,000 (including expenses incurred on
household from official account)
Insurance premium (annual) 20,000 Rs.
Existing insurance cover 6,000,0 Rs.
00
Investment yield available on 10% p.a.
investing funds till retirement
Number of remaining years to 28 years
retirement
Goal:
The anticipated increase in the Employee's post-tax salary is 5% year on year. The
employee consumes 25% of regular household expenses on self. What should be
the amount of additional insurance required to replace the Employee's
income contribution to his family for his remaining years employment?

Problem Type 5:
Situation:
Retirement age of the individual 60 yrs
Life expectancy of the individual 80 yrs
A deferred annuity pension plan offers optional one-third commutation on the date
of vesting and life certain level annuity. The level annuity from uncommuted
amount is Rs. 60,000 per month. The vesting sum of Rs. 1.5 crore is estimated on
retirement of the individual .
Goal:
If the individual opts to commute one-third of the expected vested amount and
settles for life annuity from the remaining amount, at what rate of return the
commuted amount shall be invested to yield a total income of Rs. 90,000
per month till he survives?

Problem Type 6:
Situation:
Current age of Mr. A 28 years
Current age of the spouse of Mr. A 26 years
Retirement age of Mr. A 60 years
Current house hold expenses Rs. Per month
25,000
Life expectancy of each of Mr. A and 80 years
spouse
Post-retire xpenses needed till Spouse's 75%
survival as % of pre-retire. exp.
Current investment available to be utilized Rs.
towards retirement corpus 300,000
Mr. A desires to have additional cushion of Rs. 1 crore as terminal value from the
date of last survivor towards bequest. The retirement solution can be managed at
yield of 12% p.a. in the initial 10 years, moderated to return 9% p.a. in the next 10
years. In the remaining years to retirement and continuing into retirement the funds
could be managed to yield 7% p.a. Inflation is considerd 5.5% p.a. in the pre-
retirement period and is expected to moderate at 4% p.a. in the post-retirement
period. Estimate the viability of achieving this goal by investing Rs. 62,000
p.a. in the current available investment, starting immediately, which
would be incremented by 5% in the beginning of every year. You analyze
that _______.

Problem Type 7:
Situation: Bond valuation and return

Face value of the Bond 1,000 Rs.


Coupon Rate (coupon payable at the end of every year on
31-December) 9% p.a.
Date of maturity 31-Dec-17

Current market price (as on 1-Apr-2013) 1,078 Rs.


What would be the effective return if an investment is made in the bond today
and held till maturity?

Problem Type 8:
Situation:
Client's current age 28 years
Age from which annual holidays to begin 45 years
(continuing lifelong)
Funds required at current costs 50,000 Rs. P.a.
Cost escalation for holiday expenses 7% p.a.
Age of retirement 58 years
Life expectancy 75 years
Holiday expenses to be contained post- 30,000 Rs. P.a.
retirement (at current costs) to
Goal:
The client would start investing immediately a certain amount on a quarterly basis
in investments yielding 11% p.a. up to the period of drawing expenses for holiday
for the first time. Once the corpus is built-up, the funds for holidays for 5-year block
periods would be switched to safe investments yielding 8% p.a. from which yearly
expenses would be drawn. What is the amount of quarterly investment?

Problem Type 9:
Situation:
Current household expenses of a couple 50,000 Rs. p.m.
Current age of the client 33 years
Current age of dependent spouse 30 years
Retirement age of the client 58 years
Life expectancy of the client 78 years
Life Expectany of dependent spouse 80 years
Goal:
The client wants to know the corpus required at his retirement which would be
sufficient to sustain inflation-linked monthly expenses equivalent to their pre-
retirement household expenses till the expected life of his spouse. The retirement
corpus so accumulated shall be invested at 7% p.a. return while teh ruling inflation
would be 5.5% p.a.. You estimate the corpus to be _________.

Problem Type 10:


Situation:
Current household expenses of a couple 50,000 Rs. p.m.
Current age of the client 33 years
Current age of dependent spouse 30 years
Retirement age of the client 58 years
Life expectancy of the client 78 years
Life Expectany of dependent spouse 80 years
Estimated retirement corpus the couple is 30,000,0 Rs.
confident to accumulate 00
Goal:
The client wants to now the rate of return which would see the accumuleted corpus
last till the spouse's lifetime, if inflation-linked monthly expenses are drawn at 20%
curtailmenet at the time of retirement. The inflation is considered at 5.5% p.a.

Problem Type 11:


Situation:
Your client has in his portfolio 500 shares of an unlisted company. 100 shares were
originally acquired on 1st May, 1979 for Rs. 50 each. The fair market value of the
company's share was Rs. 80 each as on 1st April, 1981. On 20th November, 1998
100 bonus shares were allotted. The fair market value was Rs. 140 each as on 20th
November, 1998. The remaining 300 shares were purchased on 30th May, 2009 for
Rs. 350 each. Your client has recently got an offer to sell the shares of the company
at Rs. 500 each. You compute the capital gains in this transaction, in case
the offer is accepted at the given price. The same is ______.

Problem Type 12:


Situation and Goal:
You observe that your client has a life insurance cover of only Rs. 20 lakh. He tells you that living e
the future 30 years for his family must be insured along with his essential goal of medical/higher e
his son and daughter. The detials are given below:
The rate of return which is expected for the funds to be 7% p.a.
invested
Inflation for living expenses 5.50% p.a.
Current household expenses p.a.
600,000
Age of son 17 years
Medical education of son to begin after a year and 5 years
required for
Current Cost of medical education Rs. p.a.
500,000
cost escalation of medical education 8% p.a.
Age of daughter 15 years
HIgher education of daughter to begin after three years 3 years
and required for
Cost of higher education for daughter Rs. p.a.
300,000
cost escalation of higher education for daughter 10% p.a.
Strategy to achieve Goal:
You suggest the client that the aggregate insurance cover should suffice to meet higher education
when due, along with inflation-adjusted living (household) expenses to the extent of 80%
current expenses for immediate next 10 years and 50% for the succeeding 20 years. Yo
the amount of insurance cover needed, which comes to _____.
Problem Type 13:
Situation:
Current age of the client 45 years
(as on 1st April 2013)
Retirement age of the 60 years
client
PPF account balance of Rs.
client as on 31-March- 500,0
2013 (maturity 1-Apr- 00
2018)
PPF a/c balance of the Rs.
spouse as on 31-March- 300,0
2013 (maturity 1-Apr- 00
2018)
Expected interest rate on 8.70% p.a. Payable:31 March: on monthly
PPF minimum balances (between 1st and
5th of every month) through the year
Goal:
As part of the retirement strategy , you advise client tp invest a sum of Rs. 40,000
and Rs. 20,000 immediately in his and spouse's account respectively and increase
this investment by 20% every year in the beginning of the financial year in all furure
years till normal maturity. The contributions are rounded up to the nearest
thousand. You also advise to extend their respective accounts for two terms of 5
years each beyond the normal maturity by contributing maximum permissible
amounts in both the accounts. Find the combined corpus of accounts when
the client retires.

Problem Type 14:


Situation:
Mr. A wants to accumulate funds for the marriage of his children, a son and a
daughter:
Present Age of Daughter 18 yrs
Age when Daughter gets married 25 yrs
Provision for Daughter's marriage Rs.
expenses at current costs 2,000,00
0
Present age of Son 16 yrs
Age when Son gets married 26 yrs
Provision for Son's marriage expenses at Rs.
current costs 1,500,00
0
Cost of escalation of marriage expenses 7.50% p.a.
Existing funds to be utilized Rs.
925,000
Goal & Strategy:
You suggest Mr. A to achieve the goal for accumulation of funds for marriage
expenses by starting a systematic regular monthly investment immediately along
with the lump sum of existing funds available in an aggressive fund for 4 years and
shift to safe investments 3 years prior to Daughter's marriage. What is the
approximate monthly investment amount required?

Problem Type 15:


Situation: Taxation of Gold Exchange
Traded Fund (Gold ETF) units
Number of units units
500
Cost of acquisition (17-Oct-2006) Rs. per unit
995
Sale Price (1-April, 2013) Rs. per unit
2,415
CII Index for the year 2006-07
519
CII Index for the year 2013-14
939
If all the units are sold at te sale price prevailing on 1st April, 2013, what
would be the post-tax gains in the transaction?

Вам также может понравиться